Animating wave motion in waterObtaining a 3D animation as a drop in a liquid surfaceChanging a wave simulationAnimating arrowsAnimating labelsAnimating a parametric plotHow to solve the tsunami model and animate the shallow water wave?Solve and design “water flow”Not getting smooth motion when animating a 3D plot (with jumps in rotation)Animating pointAnimating a 3D plot of electromagnetic wavesPlotting and Animating Spherical Spring Pendulum

It grows, but water kills it

How do you respond to a colleague from another team when they're wrongly expecting that you'll help them?

Is there a way to get `mathscr' with lower case letters in pdfLaTeX?

How can I write humor as character trait?

Hero deduces identity of a killer

What does "Scientists rise up against statistical significance" mean? (Comment in Nature)

How to fade a semiplane defined by line?

Are Captain Marvel's powers affected by Thanos' actions in Infinity War

Why is this estimator biased?

What is Cash Advance APR?

Limits and Infinite Integration by Parts

Does IPv6 have similar concept of network mask?

Do the primes contain an infinite almost arithmetic progression?

How to explain what's wrong with this application of the chain rule?

Why can Carol Danvers change her suit colours in the first place?

Using substitution ciphers to generate new alphabets in a novel

How could a planet have erratic days?

Yosemite Fire Rings - What to Expect?

Biological Blimps: Propulsion

How does a computer interpret real numbers?

How does the math work for Perception checks?

Can the US President recognize Israel’s sovereignty over the Golan Heights for the USA or does that need an act of Congress?

Recommended PCB layout understanding - ADM2572 datasheet

Can a College of Swords bard use a Blade Flourish option on an opportunity attack provoked by their own Dissonant Whispers spell?



Animating wave motion in water


Obtaining a 3D animation as a drop in a liquid surfaceChanging a wave simulationAnimating arrowsAnimating labelsAnimating a parametric plotHow to solve the tsunami model and animate the shallow water wave?Solve and design “water flow”Not getting smooth motion when animating a 3D plot (with jumps in rotation)Animating pointAnimating a 3D plot of electromagnetic wavesPlotting and Animating Spherical Spring Pendulum













22












$begingroup$


Further to this question I found on MSE, I tried to replicate



enter image description here



from here



this is as far as I got:



fun[a_, b_, c_, x_, y_] := 
Point[#[[1]] + x, #[[2]] + y &[
Part[CirclePoints[360] c,
If[a + b == 360, 360, Mod[a + b, 360]]]]];
tab = With[a = #,
Flatten[Table[
Table[fun[a, 90 + 15 n, 1 - .15 m, -1 + .5 n, -.35 m], m, 0,
10], n, 0, 24], 1]] & /@ Range[1, 360, 15];

Module[t, x, y, fun, xf, yf, a, x = -.5; y = 1;
fun[a_, b_, c_, x_, y_] :=
Point[#[[1]] + x, #[[2]] + y &[
Part[CirclePoints[360] c,
If[a + b == 360, 360, Mod[a + b, 360]]]]];
xf[t_, a_, b_] := a t - b Sin[t]; yf[t_, a_, b_] := a - b Cos[t];
Animate[
Show[
Graphics[
PointSize[.01], tab[[a]],
PlotRange -> -1 - x, 10 + x, -1 - y, 1
],
ParametricPlot[
(Pi/2) xf[t + 2 Pi a/24, 1.25, .6] - 4 Pi a/24 - Pi^2 + .05,
2.05 - 1.65 yf[t + 2 Pi a/24, 1.25, .6],
t, -4 Pi, 4 Pi, Axes -> False
]
],
a, 1, 24, 1, ControlPlacement -> Top, AnimationRate -> 5,
AnimationDirection -> Backward
]
]


which is not very efficient (I'm sure Part could be applied more efficiently), and despite various tweeks, I couldn't quite manage to get the cycloid to line up with the points.



What is a better way to approach this?










share|improve this question











$endgroup$











  • $begingroup$
    See this: mathematica.stackexchange.com/questions/123127/…
    $endgroup$
    – LCarvalho
    Mar 18 at 11:23










  • $begingroup$
    For searching purposes: this is a trochoidal wave.
    $endgroup$
    – J. M. is slightly pensive
    21 hours ago










  • $begingroup$
    @J.M.isslightlypensive thanks - looks like next step will be stokes drift!
    $endgroup$
    – martin
    20 hours ago
















22












$begingroup$


Further to this question I found on MSE, I tried to replicate



enter image description here



from here



this is as far as I got:



fun[a_, b_, c_, x_, y_] := 
Point[#[[1]] + x, #[[2]] + y &[
Part[CirclePoints[360] c,
If[a + b == 360, 360, Mod[a + b, 360]]]]];
tab = With[a = #,
Flatten[Table[
Table[fun[a, 90 + 15 n, 1 - .15 m, -1 + .5 n, -.35 m], m, 0,
10], n, 0, 24], 1]] & /@ Range[1, 360, 15];

Module[t, x, y, fun, xf, yf, a, x = -.5; y = 1;
fun[a_, b_, c_, x_, y_] :=
Point[#[[1]] + x, #[[2]] + y &[
Part[CirclePoints[360] c,
If[a + b == 360, 360, Mod[a + b, 360]]]]];
xf[t_, a_, b_] := a t - b Sin[t]; yf[t_, a_, b_] := a - b Cos[t];
Animate[
Show[
Graphics[
PointSize[.01], tab[[a]],
PlotRange -> -1 - x, 10 + x, -1 - y, 1
],
ParametricPlot[
(Pi/2) xf[t + 2 Pi a/24, 1.25, .6] - 4 Pi a/24 - Pi^2 + .05,
2.05 - 1.65 yf[t + 2 Pi a/24, 1.25, .6],
t, -4 Pi, 4 Pi, Axes -> False
]
],
a, 1, 24, 1, ControlPlacement -> Top, AnimationRate -> 5,
AnimationDirection -> Backward
]
]


which is not very efficient (I'm sure Part could be applied more efficiently), and despite various tweeks, I couldn't quite manage to get the cycloid to line up with the points.



What is a better way to approach this?










share|improve this question











$endgroup$











  • $begingroup$
    See this: mathematica.stackexchange.com/questions/123127/…
    $endgroup$
    – LCarvalho
    Mar 18 at 11:23










  • $begingroup$
    For searching purposes: this is a trochoidal wave.
    $endgroup$
    – J. M. is slightly pensive
    21 hours ago










  • $begingroup$
    @J.M.isslightlypensive thanks - looks like next step will be stokes drift!
    $endgroup$
    – martin
    20 hours ago














22












22








22


9



$begingroup$


Further to this question I found on MSE, I tried to replicate



enter image description here



from here



this is as far as I got:



fun[a_, b_, c_, x_, y_] := 
Point[#[[1]] + x, #[[2]] + y &[
Part[CirclePoints[360] c,
If[a + b == 360, 360, Mod[a + b, 360]]]]];
tab = With[a = #,
Flatten[Table[
Table[fun[a, 90 + 15 n, 1 - .15 m, -1 + .5 n, -.35 m], m, 0,
10], n, 0, 24], 1]] & /@ Range[1, 360, 15];

Module[t, x, y, fun, xf, yf, a, x = -.5; y = 1;
fun[a_, b_, c_, x_, y_] :=
Point[#[[1]] + x, #[[2]] + y &[
Part[CirclePoints[360] c,
If[a + b == 360, 360, Mod[a + b, 360]]]]];
xf[t_, a_, b_] := a t - b Sin[t]; yf[t_, a_, b_] := a - b Cos[t];
Animate[
Show[
Graphics[
PointSize[.01], tab[[a]],
PlotRange -> -1 - x, 10 + x, -1 - y, 1
],
ParametricPlot[
(Pi/2) xf[t + 2 Pi a/24, 1.25, .6] - 4 Pi a/24 - Pi^2 + .05,
2.05 - 1.65 yf[t + 2 Pi a/24, 1.25, .6],
t, -4 Pi, 4 Pi, Axes -> False
]
],
a, 1, 24, 1, ControlPlacement -> Top, AnimationRate -> 5,
AnimationDirection -> Backward
]
]


which is not very efficient (I'm sure Part could be applied more efficiently), and despite various tweeks, I couldn't quite manage to get the cycloid to line up with the points.



What is a better way to approach this?










share|improve this question











$endgroup$




Further to this question I found on MSE, I tried to replicate



enter image description here



from here



this is as far as I got:



fun[a_, b_, c_, x_, y_] := 
Point[#[[1]] + x, #[[2]] + y &[
Part[CirclePoints[360] c,
If[a + b == 360, 360, Mod[a + b, 360]]]]];
tab = With[a = #,
Flatten[Table[
Table[fun[a, 90 + 15 n, 1 - .15 m, -1 + .5 n, -.35 m], m, 0,
10], n, 0, 24], 1]] & /@ Range[1, 360, 15];

Module[t, x, y, fun, xf, yf, a, x = -.5; y = 1;
fun[a_, b_, c_, x_, y_] :=
Point[#[[1]] + x, #[[2]] + y &[
Part[CirclePoints[360] c,
If[a + b == 360, 360, Mod[a + b, 360]]]]];
xf[t_, a_, b_] := a t - b Sin[t]; yf[t_, a_, b_] := a - b Cos[t];
Animate[
Show[
Graphics[
PointSize[.01], tab[[a]],
PlotRange -> -1 - x, 10 + x, -1 - y, 1
],
ParametricPlot[
(Pi/2) xf[t + 2 Pi a/24, 1.25, .6] - 4 Pi a/24 - Pi^2 + .05,
2.05 - 1.65 yf[t + 2 Pi a/24, 1.25, .6],
t, -4 Pi, 4 Pi, Axes -> False
]
],
a, 1, 24, 1, ControlPlacement -> Top, AnimationRate -> 5,
AnimationDirection -> Backward
]
]


which is not very efficient (I'm sure Part could be applied more efficiently), and despite various tweeks, I couldn't quite manage to get the cycloid to line up with the points.



What is a better way to approach this?







performance-tuning animation physics simulation






share|improve this question















share|improve this question













share|improve this question




share|improve this question








edited 21 hours ago









J. M. is slightly pensive

98.2k10306466




98.2k10306466










asked Mar 18 at 11:08









martinmartin

4,03321249




4,03321249











  • $begingroup$
    See this: mathematica.stackexchange.com/questions/123127/…
    $endgroup$
    – LCarvalho
    Mar 18 at 11:23










  • $begingroup$
    For searching purposes: this is a trochoidal wave.
    $endgroup$
    – J. M. is slightly pensive
    21 hours ago










  • $begingroup$
    @J.M.isslightlypensive thanks - looks like next step will be stokes drift!
    $endgroup$
    – martin
    20 hours ago

















  • $begingroup$
    See this: mathematica.stackexchange.com/questions/123127/…
    $endgroup$
    – LCarvalho
    Mar 18 at 11:23










  • $begingroup$
    For searching purposes: this is a trochoidal wave.
    $endgroup$
    – J. M. is slightly pensive
    21 hours ago










  • $begingroup$
    @J.M.isslightlypensive thanks - looks like next step will be stokes drift!
    $endgroup$
    – martin
    20 hours ago
















$begingroup$
See this: mathematica.stackexchange.com/questions/123127/…
$endgroup$
– LCarvalho
Mar 18 at 11:23




$begingroup$
See this: mathematica.stackexchange.com/questions/123127/…
$endgroup$
– LCarvalho
Mar 18 at 11:23












$begingroup$
For searching purposes: this is a trochoidal wave.
$endgroup$
– J. M. is slightly pensive
21 hours ago




$begingroup$
For searching purposes: this is a trochoidal wave.
$endgroup$
– J. M. is slightly pensive
21 hours ago












$begingroup$
@J.M.isslightlypensive thanks - looks like next step will be stokes drift!
$endgroup$
– martin
20 hours ago





$begingroup$
@J.M.isslightlypensive thanks - looks like next step will be stokes drift!
$endgroup$
– martin
20 hours ago











1 Answer
1






active

oldest

votes


















31












$begingroup$

DynamicModule[t = 0, d = 5, a = .08, base, distortion, pts, r, f, n = 10,

r[y_] := .08 y^4;
f[x_] := -2 Pi Dynamic[t] + d x;
(*f does not evaluate to a number but FE will take care of that later*)

base = Array[List, n 3, 1, 0, Pi, 0, 1 ];

distortion = Array[
Function[x, y, r[y] Cos @ f @ x, Sin @ f @ x], n 3, 1, 0, Pi, 0, 1
];

pts = base + distortion;

Row[
Animator[Dynamic @ t, AnimationRate -> .8, AppearanceElements -> ],
Graphics[
LightBlue,
Polygon @ Join[ pts[[;; , -1]], Scaled[1, 0], Scaled[0, 0]],

Darker @ Blue, AbsolutePointSize @ 5, Point @ Catenate @ pts,

AbsolutePointSize @ 7, Orange, Thick,
Point @ pts[[15, -1]], Circle[base[[15, -1]], r @ base[[15, -1, 2]]],
Point @ pts[[15, 7]], Circle[base[[15, 7]], r @ base[[15, 7, 2]]]
,
PlotRange -> 0 + .1, Pi - .1, 0, 1.2,
PlotRangePadding -> 0,
PlotRangeClipping -> True, ImageSize -> 800]
]
]


enter image description here






share|improve this answer











$endgroup$












  • $begingroup$
    thanks - a vastly better approach!
    $endgroup$
    – martin
    Mar 18 at 13:34










  • $begingroup$
    @martin thanks, let me know if anything is not clear.
    $endgroup$
    – Kuba
    Mar 19 at 21:31










  • $begingroup$
    a = .08 doesn't get used; maybe you meant r[y_] := a y^4? (Also, how did you pick the form of r[y]?)
    $endgroup$
    – J. M. is slightly pensive
    14 hours ago






  • 1




    $begingroup$
    @J.M.isslightlypensive right, should be deleted. About formulas, I got them from looking at the original .gif :-) I was too lazy to think about physics etc so I assumed something like that should work. I also ignored OP's code :(
    $endgroup$
    – Kuba
    14 hours ago











Your Answer





StackExchange.ifUsing("editor", function ()
return StackExchange.using("mathjaxEditing", function ()
StackExchange.MarkdownEditor.creationCallbacks.add(function (editor, postfix)
StackExchange.mathjaxEditing.prepareWmdForMathJax(editor, postfix, [["$", "$"], ["\\(","\\)"]]);
);
);
, "mathjax-editing");

StackExchange.ready(function()
var channelOptions =
tags: "".split(" "),
id: "387"
;
initTagRenderer("".split(" "), "".split(" "), channelOptions);

StackExchange.using("externalEditor", function()
// Have to fire editor after snippets, if snippets enabled
if (StackExchange.settings.snippets.snippetsEnabled)
StackExchange.using("snippets", function()
createEditor();
);

else
createEditor();

);

function createEditor()
StackExchange.prepareEditor(
heartbeatType: 'answer',
autoActivateHeartbeat: false,
convertImagesToLinks: false,
noModals: true,
showLowRepImageUploadWarning: true,
reputationToPostImages: null,
bindNavPrevention: true,
postfix: "",
imageUploader:
brandingHtml: "Powered by u003ca class="icon-imgur-white" href="https://imgur.com/"u003eu003c/au003e",
contentPolicyHtml: "User contributions licensed under u003ca href="https://creativecommons.org/licenses/by-sa/3.0/"u003ecc by-sa 3.0 with attribution requiredu003c/au003e u003ca href="https://stackoverflow.com/legal/content-policy"u003e(content policy)u003c/au003e",
allowUrls: true
,
onDemand: true,
discardSelector: ".discard-answer"
,immediatelyShowMarkdownHelp:true
);



);













draft saved

draft discarded


















StackExchange.ready(
function ()
StackExchange.openid.initPostLogin('.new-post-login', 'https%3a%2f%2fmathematica.stackexchange.com%2fquestions%2f193480%2fanimating-wave-motion-in-water%23new-answer', 'question_page');

);

Post as a guest















Required, but never shown

























1 Answer
1






active

oldest

votes








1 Answer
1






active

oldest

votes









active

oldest

votes






active

oldest

votes









31












$begingroup$

DynamicModule[t = 0, d = 5, a = .08, base, distortion, pts, r, f, n = 10,

r[y_] := .08 y^4;
f[x_] := -2 Pi Dynamic[t] + d x;
(*f does not evaluate to a number but FE will take care of that later*)

base = Array[List, n 3, 1, 0, Pi, 0, 1 ];

distortion = Array[
Function[x, y, r[y] Cos @ f @ x, Sin @ f @ x], n 3, 1, 0, Pi, 0, 1
];

pts = base + distortion;

Row[
Animator[Dynamic @ t, AnimationRate -> .8, AppearanceElements -> ],
Graphics[
LightBlue,
Polygon @ Join[ pts[[;; , -1]], Scaled[1, 0], Scaled[0, 0]],

Darker @ Blue, AbsolutePointSize @ 5, Point @ Catenate @ pts,

AbsolutePointSize @ 7, Orange, Thick,
Point @ pts[[15, -1]], Circle[base[[15, -1]], r @ base[[15, -1, 2]]],
Point @ pts[[15, 7]], Circle[base[[15, 7]], r @ base[[15, 7, 2]]]
,
PlotRange -> 0 + .1, Pi - .1, 0, 1.2,
PlotRangePadding -> 0,
PlotRangeClipping -> True, ImageSize -> 800]
]
]


enter image description here






share|improve this answer











$endgroup$












  • $begingroup$
    thanks - a vastly better approach!
    $endgroup$
    – martin
    Mar 18 at 13:34










  • $begingroup$
    @martin thanks, let me know if anything is not clear.
    $endgroup$
    – Kuba
    Mar 19 at 21:31










  • $begingroup$
    a = .08 doesn't get used; maybe you meant r[y_] := a y^4? (Also, how did you pick the form of r[y]?)
    $endgroup$
    – J. M. is slightly pensive
    14 hours ago






  • 1




    $begingroup$
    @J.M.isslightlypensive right, should be deleted. About formulas, I got them from looking at the original .gif :-) I was too lazy to think about physics etc so I assumed something like that should work. I also ignored OP's code :(
    $endgroup$
    – Kuba
    14 hours ago
















31












$begingroup$

DynamicModule[t = 0, d = 5, a = .08, base, distortion, pts, r, f, n = 10,

r[y_] := .08 y^4;
f[x_] := -2 Pi Dynamic[t] + d x;
(*f does not evaluate to a number but FE will take care of that later*)

base = Array[List, n 3, 1, 0, Pi, 0, 1 ];

distortion = Array[
Function[x, y, r[y] Cos @ f @ x, Sin @ f @ x], n 3, 1, 0, Pi, 0, 1
];

pts = base + distortion;

Row[
Animator[Dynamic @ t, AnimationRate -> .8, AppearanceElements -> ],
Graphics[
LightBlue,
Polygon @ Join[ pts[[;; , -1]], Scaled[1, 0], Scaled[0, 0]],

Darker @ Blue, AbsolutePointSize @ 5, Point @ Catenate @ pts,

AbsolutePointSize @ 7, Orange, Thick,
Point @ pts[[15, -1]], Circle[base[[15, -1]], r @ base[[15, -1, 2]]],
Point @ pts[[15, 7]], Circle[base[[15, 7]], r @ base[[15, 7, 2]]]
,
PlotRange -> 0 + .1, Pi - .1, 0, 1.2,
PlotRangePadding -> 0,
PlotRangeClipping -> True, ImageSize -> 800]
]
]


enter image description here






share|improve this answer











$endgroup$












  • $begingroup$
    thanks - a vastly better approach!
    $endgroup$
    – martin
    Mar 18 at 13:34










  • $begingroup$
    @martin thanks, let me know if anything is not clear.
    $endgroup$
    – Kuba
    Mar 19 at 21:31










  • $begingroup$
    a = .08 doesn't get used; maybe you meant r[y_] := a y^4? (Also, how did you pick the form of r[y]?)
    $endgroup$
    – J. M. is slightly pensive
    14 hours ago






  • 1




    $begingroup$
    @J.M.isslightlypensive right, should be deleted. About formulas, I got them from looking at the original .gif :-) I was too lazy to think about physics etc so I assumed something like that should work. I also ignored OP's code :(
    $endgroup$
    – Kuba
    14 hours ago














31












31








31





$begingroup$

DynamicModule[t = 0, d = 5, a = .08, base, distortion, pts, r, f, n = 10,

r[y_] := .08 y^4;
f[x_] := -2 Pi Dynamic[t] + d x;
(*f does not evaluate to a number but FE will take care of that later*)

base = Array[List, n 3, 1, 0, Pi, 0, 1 ];

distortion = Array[
Function[x, y, r[y] Cos @ f @ x, Sin @ f @ x], n 3, 1, 0, Pi, 0, 1
];

pts = base + distortion;

Row[
Animator[Dynamic @ t, AnimationRate -> .8, AppearanceElements -> ],
Graphics[
LightBlue,
Polygon @ Join[ pts[[;; , -1]], Scaled[1, 0], Scaled[0, 0]],

Darker @ Blue, AbsolutePointSize @ 5, Point @ Catenate @ pts,

AbsolutePointSize @ 7, Orange, Thick,
Point @ pts[[15, -1]], Circle[base[[15, -1]], r @ base[[15, -1, 2]]],
Point @ pts[[15, 7]], Circle[base[[15, 7]], r @ base[[15, 7, 2]]]
,
PlotRange -> 0 + .1, Pi - .1, 0, 1.2,
PlotRangePadding -> 0,
PlotRangeClipping -> True, ImageSize -> 800]
]
]


enter image description here






share|improve this answer











$endgroup$



DynamicModule[t = 0, d = 5, a = .08, base, distortion, pts, r, f, n = 10,

r[y_] := .08 y^4;
f[x_] := -2 Pi Dynamic[t] + d x;
(*f does not evaluate to a number but FE will take care of that later*)

base = Array[List, n 3, 1, 0, Pi, 0, 1 ];

distortion = Array[
Function[x, y, r[y] Cos @ f @ x, Sin @ f @ x], n 3, 1, 0, Pi, 0, 1
];

pts = base + distortion;

Row[
Animator[Dynamic @ t, AnimationRate -> .8, AppearanceElements -> ],
Graphics[
LightBlue,
Polygon @ Join[ pts[[;; , -1]], Scaled[1, 0], Scaled[0, 0]],

Darker @ Blue, AbsolutePointSize @ 5, Point @ Catenate @ pts,

AbsolutePointSize @ 7, Orange, Thick,
Point @ pts[[15, -1]], Circle[base[[15, -1]], r @ base[[15, -1, 2]]],
Point @ pts[[15, 7]], Circle[base[[15, 7]], r @ base[[15, 7, 2]]]
,
PlotRange -> 0 + .1, Pi - .1, 0, 1.2,
PlotRangePadding -> 0,
PlotRangeClipping -> True, ImageSize -> 800]
]
]


enter image description here







share|improve this answer














share|improve this answer



share|improve this answer








edited Mar 18 at 21:35

























answered Mar 18 at 13:16









KubaKuba

107k12210530




107k12210530











  • $begingroup$
    thanks - a vastly better approach!
    $endgroup$
    – martin
    Mar 18 at 13:34










  • $begingroup$
    @martin thanks, let me know if anything is not clear.
    $endgroup$
    – Kuba
    Mar 19 at 21:31










  • $begingroup$
    a = .08 doesn't get used; maybe you meant r[y_] := a y^4? (Also, how did you pick the form of r[y]?)
    $endgroup$
    – J. M. is slightly pensive
    14 hours ago






  • 1




    $begingroup$
    @J.M.isslightlypensive right, should be deleted. About formulas, I got them from looking at the original .gif :-) I was too lazy to think about physics etc so I assumed something like that should work. I also ignored OP's code :(
    $endgroup$
    – Kuba
    14 hours ago

















  • $begingroup$
    thanks - a vastly better approach!
    $endgroup$
    – martin
    Mar 18 at 13:34










  • $begingroup$
    @martin thanks, let me know if anything is not clear.
    $endgroup$
    – Kuba
    Mar 19 at 21:31










  • $begingroup$
    a = .08 doesn't get used; maybe you meant r[y_] := a y^4? (Also, how did you pick the form of r[y]?)
    $endgroup$
    – J. M. is slightly pensive
    14 hours ago






  • 1




    $begingroup$
    @J.M.isslightlypensive right, should be deleted. About formulas, I got them from looking at the original .gif :-) I was too lazy to think about physics etc so I assumed something like that should work. I also ignored OP's code :(
    $endgroup$
    – Kuba
    14 hours ago
















$begingroup$
thanks - a vastly better approach!
$endgroup$
– martin
Mar 18 at 13:34




$begingroup$
thanks - a vastly better approach!
$endgroup$
– martin
Mar 18 at 13:34












$begingroup$
@martin thanks, let me know if anything is not clear.
$endgroup$
– Kuba
Mar 19 at 21:31




$begingroup$
@martin thanks, let me know if anything is not clear.
$endgroup$
– Kuba
Mar 19 at 21:31












$begingroup$
a = .08 doesn't get used; maybe you meant r[y_] := a y^4? (Also, how did you pick the form of r[y]?)
$endgroup$
– J. M. is slightly pensive
14 hours ago




$begingroup$
a = .08 doesn't get used; maybe you meant r[y_] := a y^4? (Also, how did you pick the form of r[y]?)
$endgroup$
– J. M. is slightly pensive
14 hours ago




1




1




$begingroup$
@J.M.isslightlypensive right, should be deleted. About formulas, I got them from looking at the original .gif :-) I was too lazy to think about physics etc so I assumed something like that should work. I also ignored OP's code :(
$endgroup$
– Kuba
14 hours ago





$begingroup$
@J.M.isslightlypensive right, should be deleted. About formulas, I got them from looking at the original .gif :-) I was too lazy to think about physics etc so I assumed something like that should work. I also ignored OP's code :(
$endgroup$
– Kuba
14 hours ago


















draft saved

draft discarded
















































Thanks for contributing an answer to Mathematica Stack Exchange!


  • Please be sure to answer the question. Provide details and share your research!

But avoid


  • Asking for help, clarification, or responding to other answers.

  • Making statements based on opinion; back them up with references or personal experience.

Use MathJax to format equations. MathJax reference.


To learn more, see our tips on writing great answers.




draft saved


draft discarded














StackExchange.ready(
function ()
StackExchange.openid.initPostLogin('.new-post-login', 'https%3a%2f%2fmathematica.stackexchange.com%2fquestions%2f193480%2fanimating-wave-motion-in-water%23new-answer', 'question_page');

);

Post as a guest















Required, but never shown





















































Required, but never shown














Required, but never shown












Required, but never shown







Required, but never shown

































Required, but never shown














Required, but never shown












Required, but never shown







Required, but never shown







Popular posts from this blog

Luettelo Yhdysvaltain laivaston lentotukialuksista Lähteet | Navigointivalikko

Adding axes to figuresAdding axes labels to LaTeX figuresLaTeX equivalent of ConTeXt buffersRotate a node but not its content: the case of the ellipse decorationHow to define the default vertical distance between nodes?TikZ scaling graphic and adjust node position and keep font sizeNumerical conditional within tikz keys?adding axes to shapesAlign axes across subfiguresAdding figures with a certain orderLine up nested tikz enviroments or how to get rid of themAdding axes labels to LaTeX figures

Gary (muusikko) Sisällysluettelo Historia | Rockin' High | Lähteet | Aiheesta muualla | NavigointivalikkoInfobox OKTuomas "Gary" Keskinen Ancaran kitaristiksiProjekti Rockin' High